Página 1 de 1

52° IMO 2011 - Problema 1

Publicado: Lun 18 Jul, 2011 8:35 pm
por Caro - V3
Para cualquier conjunto [math] de cuatro enteros positivos distintos se denota la suma [math] por [math]. Sea [math] el número de parejas [math] con [math] para las cuales [math] divide a [math]. Encontrar todos los conjuntos [math] de cuatro enteros positivos distintos para los cuales se alcanza el mayor valor posible de [math].

Re: 52° IMO 2011 - Problema 1

Publicado: Mar 19 Jul, 2011 12:28 am
por Vladislao
Creo que no me faltó nada.
Spoiler: mostrar
Para facilitarnos el trámite, pongamos [math], [math], [math] y [math].

Entonces, veamos que [math].

En efecto, pues, si ponemos [math], entonces, [math], por lo que [math].

Análogamente, [math], y entonces [math]

Como hay exactamente 6 pares posibles [math] con [math] y hay dos que no cumplen la condición del enunciado, entonces [math] como queríamos.

Vamos a hallar todos los conjuntos tales que [math].

Tenemos que:

[math]
[math]
[math]
[math]

De las últimas dos relaciones, obtenemos que [math], o lo que es equivalente, [math]. Por lo que, pongamos [math], y [math].

Entonces, la segunda relación equivale a que:

[math]

Reemplazándola en la primera:

[math]

Esto es:

[math]

[math]

Entonces, [math], de donde [math].

i) Si [math], entonces:

[math]

Entonces, hay tres posibilidades.

1) [math], lo cual es absurdo.
2) [math], lo cual es una contradicción.
3) [math]

Si [math], entonces como [math], entonces [math]

Como [math], y [math], entonces [math], absurdo. Luego no hay conjuntos para [math].

ii) Si [math], entonces:

[math]

Entonces, nuevamente, hay tres posibilidades:

1) [math], absurdo.
2) [math], contradicción.
3) [math]

Si [math], entonces se tiene que [math], de donde [math]

Entonces [math], pero [math], absurdo. No hay conjuntos para [math].

iii) Si [math], entonces:

[math]

Entonces, existe un [math] entero positivo tal que [math], y entonces [math].

Si [math], [math], imposible.

Si [math]. Como [math], entonces [math], [math], entonces [math].

Si [math]. Como [math], entonces [math], [math], entonces [math]

Si [math]. Como [math], entonces [math], [math], entonces [math], pero [math] es decir, [math], absurdo.

Entonces, todos los conjuntos posibles, son los de la forma:

[math] y [math]

Re: 52° IMO 2011 - Problema 1

Publicado: Mar 19 Jul, 2011 1:19 am
por sebach
Algo que saqué es esto:
Spoiler: mostrar
Digamos que [math] dividen a [math]. Luego, [math].
Como queremos la mayor cantidad, digamos que [math] también divide a [math]. Luego, [math].
[math].
Unica posibilidad [math]. Entonces, [math]. [math]
Ahora, digamos que hay otras dos parejas (formadas por números distintos) con las que se cumple esto, por ejemplo, [math]. [math]
Ahora resto [math] con [math] y tengo que [math] . Pero el problema pide que todos los número sean distintos.
Entonces, hay a lo sumo dos parejas que sumen los mismo.
Entonces, supongamos que [math], [math], [math] y [math] dividen a [math]. [math] no pueden dividir a [math], ya que [math] dividen a [math], y ya tenemos las dos parejas que suman lo mismo. [math] tampoco puede divir a [math].
Y así tenemos que a lo sumo cuatro parejas dividen a [math].
Ahora no sé como seguir...

Re: 52° IMO 2011 - Problema 1

Publicado: Mar 30 Abr, 2019 1:05 pm
por BrunZo
Solución: (sin partes)
Spoiler: mostrar
$a_i+a_j\mid s_A\Longleftrightarrow a_i+a_j\mid a_k+a_m$ donde $k$ y $m$ son los índices faltantes.
Sean $a_1<a_2<a_3<a_4$.
Como
  • $a_2+a_4>a_1+a_3$.
  • $a_3+a_4>a_1+a_2$.
Tenemos $n_A\leq 4$. Más aún, si $n_A=4$, $a_1+a_4=a_2+a_3$.
Vamos a buscar los casos con $n_A=4$. Sean $a<b<c<b+c-a$ los números. Entonces
$$a+b\mid b+2c-a\Longrightarrow a+b\mid 2(b+c)$$
$$a+c\mid 2b+c-a\Longrightarrow a+c\mid 2(b+c)$$
Sea $b+c=n$. Nosotros queremos determinar si existen $b+c=n$ con $b<c$ (1) tal que exista un divisor $d$ de $2n$ con $b<d=a+b<2b$ (2) tal que $d'=d+(c-b)=a+c$ sea otro divisor.
De (1) se sigue que $\frac{n}{2}<c$.
De (2) se sigue que $c<d'<b+c=n$.
De estas últimas dos se sigue
$\frac{2n}{4}=\frac{n}{2}<c<d'<n=\frac{2n}{2}\Longrightarrow d'=\frac{2n}{3}$ ya que $d'$ es divisor de $2n$ entre $\frac{2n}{4}$ y $\frac{2n}{2}$.
De $c<d'=\frac{2n}{3}$ se sigue que
$\frac{2n}{6}<b<d<\frac{2n}{3}\Longrightarrow d=\frac{2n}{5}\vee d=\frac{2n}{4}$ ya que $d'$ es divisor de $2n$ entre $\frac{2n}{6}$ y $\frac{2n}{4}$.
Entonces, como $c-b=d'-d$, tenemos que $c-b=\frac{4n}{15}\vee c-b=\frac{n}{6}$ de lo que se sigue $b=\frac{11n}{30}\vee b=\frac{5n}{12}$.
De $a=d-b$ se sigue que $a=\frac{n}{30}\vee a=\frac{n}{12}$ para los respectivos casos.
Entonces, tenemos que $(a,b,c)=(\frac{n}{30},\frac{11n}{30},\frac{19n}{30})$ $(a,b,c)=(\frac{n}{12},\frac{5n}{12},\frac{7n}{12})$.
Considerando que los números son enteros, tenemos las parametrizaciones:
$$(a,11a,19a,29a),\quad (a,5a,7a,11a)$$
que funcionan.

Re: 52° IMO 2011 - Problema 1

Publicado: Lun 24 May, 2021 10:24 am
por LorenzoRD
Spoiler: mostrar
Primero, vemos que hay $\binom{4}{2} = 6$ formas de elegir pares de elementos de $A$. Supongamos, sin perder generalidad, que $a_1 < a_2 < a_3 < a_4$.
Notemos que (siendo $A = \{a_i, a_j, a_k, a_l\}$), $a_i + a_j \mid s_A \iff a_i + a_j \mid a_i + a_j + a_k + a_l \iff a_i + a_j \mid a_k + a_l \Rightarrow a_i + a_j ≤ a_k + a_l$.
Del mismo modo, $a_k + a_l \mid s_A \Rightarrow a_k + a_l ≤ a_i + a_j$.
Por lo que la única forma de que se cumplan a la vez $a_i + a_j \mid s_A \wedge a_k + a_l \mid s_A$ es que $a_i + a_j = a_k + a_l$.

Notemos que como $a_3 > a_1 \wedge a_4 > a_2$, entonces $a_3 + a_4 > a_1 + a_2$.
Del mismo modo, como $a_4 > a_3 \wedge a_2 > a_1$, entonces $a_4 + a_2 > a_3 + a_1$.
Por lo que solamente podría pasar $a_1 + a_4 = a_2 + a_3$, entonces $n_A ≤ 4$.
Para ver que el máximo es $4$, podemos ver un ejemplo que claramente funciona, $A = \{1, 5, 7, 11\}$.


Ahora pasamos a la segunda parte del problema, hallar todos los conjuntos $A$ que cumplan las condiciones del enunciado. Ya encontramos una condición clave, que es $a_1 + a_4 = a_2 + a_3$. Y como $a_1 + a_2 + a_3 + a_4 = s_A$, ambos lados de la ecuación anterior serian iguales a $\frac{s_A}{2}$, por lo que dividen a $s_A$ y el cociente es $2$.

Dado que $a_3 + a_4 > a_1 + a_2 \wedge a_4 + a_2 > a_3 + a_1$, tenemos $a_3 + a_4 > \frac{s_A}{2} \wedge a_4 + a_2 > \frac{s_A}{2}$. Por lo que las divisiones que pueden ser enteras son con denominador $a_1 + a_2 \wedge a_1 + a_3$.
Vamos a usar también que $a_4 = a_2 + a_3 - a_1$.

$a_1 + a_2 \mid a_1 + a_2 + a_3 + a_2 + a_3 - a_1 \iff a_1 + a_2 \mid 2(a_2 + a_3)$
$a_1 + a_3 \mid a_1 + a_2 + a_3 + a_2 + a_3 - a_1 \iff a_1 + a_3 \mid 2(a_2 + a_3)$

Supongamos que estas dos divisiones son enteras, y sean $x$ e $y$ los cocientes respectivamente. Como $a_1 + a_2 < a_1 + a_3$, $x > y$.
Tenemos que $y(a_1 + a_3) = 2(a_2 + a_3)$.
Si $y ≤ 2$, entonces: $y(a_1 + a_3) ≤ 2(a_1 + a_3) < 2(a_2 + a_3)$.
Si $y ≥ 4$, entonces: $y(a_1 + a_3) ≥ 4(a_1 + a_3) > 2(a_2 + a_3)$ (en este, ya que cancelando los $a_3$ quedan igual o más cantidad de $a_3$ en el lado izquierdo que de $a_2$ en el lado derecho, y $a_3 > a_2$).
Por lo que $y = 3$.
Esto implica que $3(a_1 + a_3) = 2(a_2 + a_3)$, por lo que $a_3 = 2a_2 - 3a_1$.

Ahora con $x$. Tenemos que $x > 3 \wedge x(a_1 + a_2) = 2(a_2 + a_3)$, de donde $xa_1 + (x - 2)a_2 = 2a_3$
Reemplazando el valor de $a_3$ de antes, queda $xa_1 + (x - 2)a_2 = 4a_2 - 6a_1$, y finalmente $(x + 6)a_1 = (6 - x)a_2$
Notemos que el lado izquierdo es positivo, ya que $x > 3$ y por enunciado los elementos de $A$ son positivos. Luego $x < 6$.

Tenemos dos casos, que son $x = 4 \wedge x = 5$.
Caso $x = 4$, en esa ecuación $5a_1 = a_2$, luego con la de $a_3 = 2a_2 - 3a_1$ se llega a $a_3 = 7a_1$ y finalmente la de $a_4 = a_2 + a_3 - a_1$ se llega a $a_4 = 11a_1$.
Caso $x = 5$, en esa ecuación $11a_1 = a_2$, luego con la de $a_3 = 2a_2 - 3a_1$ se llega a $a_3 = 19a_1$ y finalmente la de $a_4 = a_2 + a_3 - a_1$ se llega a $a_4 = 29a_1$.

Por lo que las soluciones son $A = \{a, 5a, 7a, 11a\}$, $A = \{a, 11a, 19a, 29a\}$ con $a$ entero positivo y considerando todas las permutaciones.